LSAT and Law School Admissions Forum

Get expert LSAT preparation and law school admissions advice from PowerScore Test Preparation.

 gen2871
  • Posts: 47
  • Joined: Jul 01, 2018
|
#49353
Coolio. thank you!
 Dianapoo
  • Posts: 24
  • Joined: Sep 15, 2018
|
#58052
Hi!

I was wondering if A would have been a Good answer if it said:

mistake a condition that sometimes accompanies unsafe smog with one that is SUFFFICIENT for it.

Say the sun is sufficent for smog: sun --> smog

The other stays the same: wind blows <-s-> sun

Put these together: wind blows <-s-> sun --> smog

Use Dave's some train and boom wind blows <-s-> smog

Now we can see why the author came to that conclusion! Thus my version of A would be an answer?

Thank you! I'm trying to get a little creative :-D
 Adam Brinker
PowerScore Staff
  • PowerScore Staff
  • Posts: 42
  • Joined: Mar 28, 2011
|
#58057
Hi Diana,

Does the explanation of this question here help:
lsat/viewtopic.php?t=3463

Please let us know. Thanks!
 Dianapoo
  • Posts: 24
  • Joined: Sep 15, 2018
|
#58080
Adam Brinker wrote:Hi Diana,

Does the explanation of this question here help:
lsat/viewtopic.php?t=3463

Please let us know. Thanks!
Hi Adam!

I totally get the answer, I was just seeing if my approach to a slightly altered question would have been good!
 Adam Tyson
PowerScore Staff
  • PowerScore Staff
  • Posts: 5153
  • Joined: Apr 14, 2011
|
#58547
Yes, Diana, that would have worked, and your breakdown is correct. If "hot days" was sufficient for unsafe smog, and there were some hot days that also had the wind blowing from the east, then on at least those hot days there would be both unsafe smog and wind from the east, and the author's conclusion would have been correct rather than flawed. Nicely done!
 lsatfighter
  • Posts: 26
  • Joined: Sep 26, 2018
|
#65629
I have read the explanation of this question from the LR Bible. I understand why B is right, but I have some questions about E.

Is it true that there is no evidence of a correlation between smog and wind blowing? That is, since we don't know that those two occur on the same day, there is no evidence of a correlation? And if those two did occur on the same day, would that be considered a correlation?

If someone could answer those questions and further elaborate on why E is wrong, I'd appreciate it.
 lsatfighter
  • Posts: 26
  • Joined: Sep 26, 2018
|
#65967
Hello. Unfortunately, I have not received a response to my question. If somebody could please help me out, I'd appreciate it. Thank you in advance.
 Brook Miscoski
PowerScore Staff
  • PowerScore Staff
  • Posts: 418
  • Joined: Sep 13, 2018
|
#66132
lsatfighter,

When you see a standard flaw on the LSAT and are asked to identify the flaw, you should select the choice that responds directly to the standard flaw.

In this case, there is a formal logic flaw because you cannot chain two "some" statements together. You should be familiar with that prohibition and the reason for it, but in review:

Some houses are white.
Some houses are brick.

Therefore there are some white brick houses.

The problem is made simple by assigning numbers, remembering that "some" means "at least one":

1 of 10 total houses is white.
1 of 10 total houses is brick.

Now it is easy to see that there is no reason why the white house should also be a brick house. There are plenty of houses to spread those characteristics around without any overlap. Note that these are not probability claims where, given enough houses, it might be unreasonable to think that a 1/10 probability of white and 1/10 probability of brick never result in a house with both characteristics. It's just 10 houses, 1 is white, and 1 is brick. It could be 1 white brick house and 9 other houses, or it could be 1 white, 1 brick, and 8 other houses. Concluding there is definitely overlap is flawed.

Looking at answer choice (B), that choice directly states the problem just illustrated. That is ample proof that (B) is the credited response.

The reason why (E) is wrong is that there is nothing in the stimulus that would indicate that the stimulus is making a causal argument or one based on statistical correlation. It simply made a formal logic error by chaining "some" statements together. The reason why (E) is attractive to you is that you probably assume that there could be some kind of causal connection or statistical correlation between hot days, smog, and wind. However, you have to be careful and confirm whether the stimulus uses causal words or correlation words to associate these things, and it doesn't. They are simply things that happen.

Get the most out of your LSAT Prep Plus subscription.

Analyze and track your performance with our Testing and Analytics Package.